Integral Operator in $L^2$












4












$begingroup$


I was trying to do this exercise and I'm wondering if I figured it out well:



I have $mathcal{H} := L^2(0,1)$ and $T$ the operator with integral kernel $K(x,y) = min{x,y}$, $x,y in [0,1]$. I have to show that $T$ is compact and self-adjoint.



To show that is compact I was thinking to say that because $min{x,y} in [0,1]$ then



begin{equation}
dim(operatorname{Im}T) = 1
end{equation}



(The self adjointness I think is trivial..)So T belongs to finite rank operators and so it is compact. (Is this correct?) Then it asks me to find eigenvalues and eigenvectors of $T$ and here I really don't know how to proceed...










share|cite|improve this question











$endgroup$












  • $begingroup$
    When you claim the dimension is 1, you mean to claim that there is a function $f$ such that for any function $gin L^2$, there is a constant $lambda=lambda(g)$ such that $int_0^1 K(x,y)g(y)dy = lambda f(x)$?
    $endgroup$
    – Calvin Khor
    Jan 5 at 19:17










  • $begingroup$
    Actually I meant that $ImT = < 1 >$. But I don't know if it's right..
    $endgroup$
    – James Arten
    Jan 5 at 19:20












  • $begingroup$
    So I think in what I wrote, you'd be claiming that the function $f$ is identically 1. No, this is not true, if you take $g=1in L^2$ then $int_0^1 min(x,y) cdot 1 dy $ is not constant in $x$.
    $endgroup$
    – Calvin Khor
    Jan 5 at 19:23










  • $begingroup$
    Yea sorry I didn't mean < 1 > because if I consider $int_0^{1} min{x,y}f(y),dy$ it will be a function of $x$. So the $ImT$ will be generated by all possible linear combinations of $x$, is this right?
    $endgroup$
    – James Arten
    Jan 5 at 19:27






  • 1




    $begingroup$
    Please observe in desmos.com/calculator that $T1$ is not linear
    $endgroup$
    – Calvin Khor
    Jan 5 at 19:27


















4












$begingroup$


I was trying to do this exercise and I'm wondering if I figured it out well:



I have $mathcal{H} := L^2(0,1)$ and $T$ the operator with integral kernel $K(x,y) = min{x,y}$, $x,y in [0,1]$. I have to show that $T$ is compact and self-adjoint.



To show that is compact I was thinking to say that because $min{x,y} in [0,1]$ then



begin{equation}
dim(operatorname{Im}T) = 1
end{equation}



(The self adjointness I think is trivial..)So T belongs to finite rank operators and so it is compact. (Is this correct?) Then it asks me to find eigenvalues and eigenvectors of $T$ and here I really don't know how to proceed...










share|cite|improve this question











$endgroup$












  • $begingroup$
    When you claim the dimension is 1, you mean to claim that there is a function $f$ such that for any function $gin L^2$, there is a constant $lambda=lambda(g)$ such that $int_0^1 K(x,y)g(y)dy = lambda f(x)$?
    $endgroup$
    – Calvin Khor
    Jan 5 at 19:17










  • $begingroup$
    Actually I meant that $ImT = < 1 >$. But I don't know if it's right..
    $endgroup$
    – James Arten
    Jan 5 at 19:20












  • $begingroup$
    So I think in what I wrote, you'd be claiming that the function $f$ is identically 1. No, this is not true, if you take $g=1in L^2$ then $int_0^1 min(x,y) cdot 1 dy $ is not constant in $x$.
    $endgroup$
    – Calvin Khor
    Jan 5 at 19:23










  • $begingroup$
    Yea sorry I didn't mean < 1 > because if I consider $int_0^{1} min{x,y}f(y),dy$ it will be a function of $x$. So the $ImT$ will be generated by all possible linear combinations of $x$, is this right?
    $endgroup$
    – James Arten
    Jan 5 at 19:27






  • 1




    $begingroup$
    Please observe in desmos.com/calculator that $T1$ is not linear
    $endgroup$
    – Calvin Khor
    Jan 5 at 19:27
















4












4








4





$begingroup$


I was trying to do this exercise and I'm wondering if I figured it out well:



I have $mathcal{H} := L^2(0,1)$ and $T$ the operator with integral kernel $K(x,y) = min{x,y}$, $x,y in [0,1]$. I have to show that $T$ is compact and self-adjoint.



To show that is compact I was thinking to say that because $min{x,y} in [0,1]$ then



begin{equation}
dim(operatorname{Im}T) = 1
end{equation}



(The self adjointness I think is trivial..)So T belongs to finite rank operators and so it is compact. (Is this correct?) Then it asks me to find eigenvalues and eigenvectors of $T$ and here I really don't know how to proceed...










share|cite|improve this question











$endgroup$




I was trying to do this exercise and I'm wondering if I figured it out well:



I have $mathcal{H} := L^2(0,1)$ and $T$ the operator with integral kernel $K(x,y) = min{x,y}$, $x,y in [0,1]$. I have to show that $T$ is compact and self-adjoint.



To show that is compact I was thinking to say that because $min{x,y} in [0,1]$ then



begin{equation}
dim(operatorname{Im}T) = 1
end{equation}



(The self adjointness I think is trivial..)So T belongs to finite rank operators and so it is compact. (Is this correct?) Then it asks me to find eigenvalues and eigenvectors of $T$ and here I really don't know how to proceed...







functional-analysis eigenvalues-eigenvectors compact-operators






share|cite|improve this question















share|cite|improve this question













share|cite|improve this question




share|cite|improve this question








edited Jan 5 at 19:17









Davide Giraudo

126k16150261




126k16150261










asked Jan 5 at 19:13









James ArtenJames Arten

12911




12911












  • $begingroup$
    When you claim the dimension is 1, you mean to claim that there is a function $f$ such that for any function $gin L^2$, there is a constant $lambda=lambda(g)$ such that $int_0^1 K(x,y)g(y)dy = lambda f(x)$?
    $endgroup$
    – Calvin Khor
    Jan 5 at 19:17










  • $begingroup$
    Actually I meant that $ImT = < 1 >$. But I don't know if it's right..
    $endgroup$
    – James Arten
    Jan 5 at 19:20












  • $begingroup$
    So I think in what I wrote, you'd be claiming that the function $f$ is identically 1. No, this is not true, if you take $g=1in L^2$ then $int_0^1 min(x,y) cdot 1 dy $ is not constant in $x$.
    $endgroup$
    – Calvin Khor
    Jan 5 at 19:23










  • $begingroup$
    Yea sorry I didn't mean < 1 > because if I consider $int_0^{1} min{x,y}f(y),dy$ it will be a function of $x$. So the $ImT$ will be generated by all possible linear combinations of $x$, is this right?
    $endgroup$
    – James Arten
    Jan 5 at 19:27






  • 1




    $begingroup$
    Please observe in desmos.com/calculator that $T1$ is not linear
    $endgroup$
    – Calvin Khor
    Jan 5 at 19:27




















  • $begingroup$
    When you claim the dimension is 1, you mean to claim that there is a function $f$ such that for any function $gin L^2$, there is a constant $lambda=lambda(g)$ such that $int_0^1 K(x,y)g(y)dy = lambda f(x)$?
    $endgroup$
    – Calvin Khor
    Jan 5 at 19:17










  • $begingroup$
    Actually I meant that $ImT = < 1 >$. But I don't know if it's right..
    $endgroup$
    – James Arten
    Jan 5 at 19:20












  • $begingroup$
    So I think in what I wrote, you'd be claiming that the function $f$ is identically 1. No, this is not true, if you take $g=1in L^2$ then $int_0^1 min(x,y) cdot 1 dy $ is not constant in $x$.
    $endgroup$
    – Calvin Khor
    Jan 5 at 19:23










  • $begingroup$
    Yea sorry I didn't mean < 1 > because if I consider $int_0^{1} min{x,y}f(y),dy$ it will be a function of $x$. So the $ImT$ will be generated by all possible linear combinations of $x$, is this right?
    $endgroup$
    – James Arten
    Jan 5 at 19:27






  • 1




    $begingroup$
    Please observe in desmos.com/calculator that $T1$ is not linear
    $endgroup$
    – Calvin Khor
    Jan 5 at 19:27


















$begingroup$
When you claim the dimension is 1, you mean to claim that there is a function $f$ such that for any function $gin L^2$, there is a constant $lambda=lambda(g)$ such that $int_0^1 K(x,y)g(y)dy = lambda f(x)$?
$endgroup$
– Calvin Khor
Jan 5 at 19:17




$begingroup$
When you claim the dimension is 1, you mean to claim that there is a function $f$ such that for any function $gin L^2$, there is a constant $lambda=lambda(g)$ such that $int_0^1 K(x,y)g(y)dy = lambda f(x)$?
$endgroup$
– Calvin Khor
Jan 5 at 19:17












$begingroup$
Actually I meant that $ImT = < 1 >$. But I don't know if it's right..
$endgroup$
– James Arten
Jan 5 at 19:20






$begingroup$
Actually I meant that $ImT = < 1 >$. But I don't know if it's right..
$endgroup$
– James Arten
Jan 5 at 19:20














$begingroup$
So I think in what I wrote, you'd be claiming that the function $f$ is identically 1. No, this is not true, if you take $g=1in L^2$ then $int_0^1 min(x,y) cdot 1 dy $ is not constant in $x$.
$endgroup$
– Calvin Khor
Jan 5 at 19:23




$begingroup$
So I think in what I wrote, you'd be claiming that the function $f$ is identically 1. No, this is not true, if you take $g=1in L^2$ then $int_0^1 min(x,y) cdot 1 dy $ is not constant in $x$.
$endgroup$
– Calvin Khor
Jan 5 at 19:23












$begingroup$
Yea sorry I didn't mean < 1 > because if I consider $int_0^{1} min{x,y}f(y),dy$ it will be a function of $x$. So the $ImT$ will be generated by all possible linear combinations of $x$, is this right?
$endgroup$
– James Arten
Jan 5 at 19:27




$begingroup$
Yea sorry I didn't mean < 1 > because if I consider $int_0^{1} min{x,y}f(y),dy$ it will be a function of $x$. So the $ImT$ will be generated by all possible linear combinations of $x$, is this right?
$endgroup$
– James Arten
Jan 5 at 19:27




1




1




$begingroup$
Please observe in desmos.com/calculator that $T1$ is not linear
$endgroup$
– Calvin Khor
Jan 5 at 19:27






$begingroup$
Please observe in desmos.com/calculator that $T1$ is not linear
$endgroup$
– Calvin Khor
Jan 5 at 19:27












3 Answers
3






active

oldest

votes


















2












$begingroup$

We have
$$
int_{(0,1)^2} |k(x,y)|^2 mathsf d(xtimes y) = int_0^1int_0^1 (xwedge y)^2 mathsf dx mathsf dy
leqslant int_0^1int_0^1 mathsf dx mathsf dy = 1 <infty,
$$

so T is a Hilbert-Schmidt operator and hence is compact.






share|cite|improve this answer









$endgroup$













  • $begingroup$
    Thanks! What about eigenvalues/eigenvectors?
    $endgroup$
    – James Arten
    Jan 5 at 20:42



















1












$begingroup$

$T$ is a Hibert-Schmidt operator because $min{x,y} in L^2([0,1]times[0,1])$. $Tf$ may be written as
begin{align}
Tf & = int_{0}^{1}K(x,y)f(y)dy \
& = int_{0}^{1}min{x,y}f(y)dy \
& = int_0^xyf(y)dy+xint_x^1 f(y)dy
end{align}

If $Tf=lambda f$ for some $fin L^2$ and $lambdainmathbb{C}setminus{0}$, then the above implies that $Tf$ is equal a.e. to a continuous function on $[0,1]$. Hence, $f$ is equal a.e. to a continuous function. So $Tf$ is continuously differentiable. So, assume without loss of generality, that $f$ is continuous. Then $(Tf)(0)=0$ and $(Tf)'$ exists with
$$
lambda f'= (Tf)'=xf(x)-xf(x)+int_x^1f(y)dty=int_x^1 f(y)dt
$$

So $f$ is $C^2$, $f(0)=0$, $f'(1)=0$ and $lambda f''=-f$ for every eigenfunction with eigenvalue $lambdane 0$. The eigenfunctions with non-zero eigenvalues are, therefore, constant multiplies of
$$
f_n = sin(npi x/2),;;; n=1,3,5,7,cdots, \
lambda_n = frac{2}{npi}.
$$



The adjoint of $int_0^x$ is $int_x^1$. And the adjoint of $M_x$ (multiplication by $x$) is $M_x$. So $T$ is selfadjoint because
begin{align}
T &= left(int_0^xright)M_x+M_xleft(int_x^1right)\ &=left(int_0^xright)M_x+M_x^*left(int_0^xright)^* = T^*.end{align}






share|cite|improve this answer









$endgroup$





















    0












    $begingroup$

    Since we have the integral identity
    $$ g:= Tf = int_0^x yf(y) dy + xint_x^1 f(y) dy $$
    Then $gin H^1$, and since $g' = int_x^1 f(y) dy$, actually $g in H^2$, with
    $$ -g'' = f$$
    So we actually have a Poisson equation, but with boundary conditions $g(0)=0,g(1) = int_0^1 yf(y) dy$. By setting $tilde g = g - x int_0^1 yf(y) dy$, we notice that we are equivalently looking for the weak solutions in $H^1_0$ to the Poisson equation with Dirichlet boundary conditions
    $$ -tilde g'' = f text{ on } (0,1),quad tilde g(0)=tilde g(1)=0,quad fin L^2 $$
    So if you already knew that the solution operator $fmapsto tilde g$ for this 1D Poisson equation was compact($H^1_0 subsetsubset L^2)$ and self-adjoint, we're done (the difference $ g-tilde g= xint_0^1 y f(y) dy$ is finite rank and self-adjoint).






    share|cite|improve this answer









    $endgroup$













      Your Answer





      StackExchange.ifUsing("editor", function () {
      return StackExchange.using("mathjaxEditing", function () {
      StackExchange.MarkdownEditor.creationCallbacks.add(function (editor, postfix) {
      StackExchange.mathjaxEditing.prepareWmdForMathJax(editor, postfix, [["$", "$"], ["\\(","\\)"]]);
      });
      });
      }, "mathjax-editing");

      StackExchange.ready(function() {
      var channelOptions = {
      tags: "".split(" "),
      id: "69"
      };
      initTagRenderer("".split(" "), "".split(" "), channelOptions);

      StackExchange.using("externalEditor", function() {
      // Have to fire editor after snippets, if snippets enabled
      if (StackExchange.settings.snippets.snippetsEnabled) {
      StackExchange.using("snippets", function() {
      createEditor();
      });
      }
      else {
      createEditor();
      }
      });

      function createEditor() {
      StackExchange.prepareEditor({
      heartbeatType: 'answer',
      autoActivateHeartbeat: false,
      convertImagesToLinks: true,
      noModals: true,
      showLowRepImageUploadWarning: true,
      reputationToPostImages: 10,
      bindNavPrevention: true,
      postfix: "",
      imageUploader: {
      brandingHtml: "Powered by u003ca class="icon-imgur-white" href="https://imgur.com/"u003eu003c/au003e",
      contentPolicyHtml: "User contributions licensed under u003ca href="https://creativecommons.org/licenses/by-sa/3.0/"u003ecc by-sa 3.0 with attribution requiredu003c/au003e u003ca href="https://stackoverflow.com/legal/content-policy"u003e(content policy)u003c/au003e",
      allowUrls: true
      },
      noCode: true, onDemand: true,
      discardSelector: ".discard-answer"
      ,immediatelyShowMarkdownHelp:true
      });


      }
      });














      draft saved

      draft discarded


















      StackExchange.ready(
      function () {
      StackExchange.openid.initPostLogin('.new-post-login', 'https%3a%2f%2fmath.stackexchange.com%2fquestions%2f3063087%2fintegral-operator-in-l2%23new-answer', 'question_page');
      }
      );

      Post as a guest















      Required, but never shown

























      3 Answers
      3






      active

      oldest

      votes








      3 Answers
      3






      active

      oldest

      votes









      active

      oldest

      votes






      active

      oldest

      votes









      2












      $begingroup$

      We have
      $$
      int_{(0,1)^2} |k(x,y)|^2 mathsf d(xtimes y) = int_0^1int_0^1 (xwedge y)^2 mathsf dx mathsf dy
      leqslant int_0^1int_0^1 mathsf dx mathsf dy = 1 <infty,
      $$

      so T is a Hilbert-Schmidt operator and hence is compact.






      share|cite|improve this answer









      $endgroup$













      • $begingroup$
        Thanks! What about eigenvalues/eigenvectors?
        $endgroup$
        – James Arten
        Jan 5 at 20:42
















      2












      $begingroup$

      We have
      $$
      int_{(0,1)^2} |k(x,y)|^2 mathsf d(xtimes y) = int_0^1int_0^1 (xwedge y)^2 mathsf dx mathsf dy
      leqslant int_0^1int_0^1 mathsf dx mathsf dy = 1 <infty,
      $$

      so T is a Hilbert-Schmidt operator and hence is compact.






      share|cite|improve this answer









      $endgroup$













      • $begingroup$
        Thanks! What about eigenvalues/eigenvectors?
        $endgroup$
        – James Arten
        Jan 5 at 20:42














      2












      2








      2





      $begingroup$

      We have
      $$
      int_{(0,1)^2} |k(x,y)|^2 mathsf d(xtimes y) = int_0^1int_0^1 (xwedge y)^2 mathsf dx mathsf dy
      leqslant int_0^1int_0^1 mathsf dx mathsf dy = 1 <infty,
      $$

      so T is a Hilbert-Schmidt operator and hence is compact.






      share|cite|improve this answer









      $endgroup$



      We have
      $$
      int_{(0,1)^2} |k(x,y)|^2 mathsf d(xtimes y) = int_0^1int_0^1 (xwedge y)^2 mathsf dx mathsf dy
      leqslant int_0^1int_0^1 mathsf dx mathsf dy = 1 <infty,
      $$

      so T is a Hilbert-Schmidt operator and hence is compact.







      share|cite|improve this answer












      share|cite|improve this answer



      share|cite|improve this answer










      answered Jan 5 at 20:40









      Math1000Math1000

      19.1k31745




      19.1k31745












      • $begingroup$
        Thanks! What about eigenvalues/eigenvectors?
        $endgroup$
        – James Arten
        Jan 5 at 20:42


















      • $begingroup$
        Thanks! What about eigenvalues/eigenvectors?
        $endgroup$
        – James Arten
        Jan 5 at 20:42
















      $begingroup$
      Thanks! What about eigenvalues/eigenvectors?
      $endgroup$
      – James Arten
      Jan 5 at 20:42




      $begingroup$
      Thanks! What about eigenvalues/eigenvectors?
      $endgroup$
      – James Arten
      Jan 5 at 20:42











      1












      $begingroup$

      $T$ is a Hibert-Schmidt operator because $min{x,y} in L^2([0,1]times[0,1])$. $Tf$ may be written as
      begin{align}
      Tf & = int_{0}^{1}K(x,y)f(y)dy \
      & = int_{0}^{1}min{x,y}f(y)dy \
      & = int_0^xyf(y)dy+xint_x^1 f(y)dy
      end{align}

      If $Tf=lambda f$ for some $fin L^2$ and $lambdainmathbb{C}setminus{0}$, then the above implies that $Tf$ is equal a.e. to a continuous function on $[0,1]$. Hence, $f$ is equal a.e. to a continuous function. So $Tf$ is continuously differentiable. So, assume without loss of generality, that $f$ is continuous. Then $(Tf)(0)=0$ and $(Tf)'$ exists with
      $$
      lambda f'= (Tf)'=xf(x)-xf(x)+int_x^1f(y)dty=int_x^1 f(y)dt
      $$

      So $f$ is $C^2$, $f(0)=0$, $f'(1)=0$ and $lambda f''=-f$ for every eigenfunction with eigenvalue $lambdane 0$. The eigenfunctions with non-zero eigenvalues are, therefore, constant multiplies of
      $$
      f_n = sin(npi x/2),;;; n=1,3,5,7,cdots, \
      lambda_n = frac{2}{npi}.
      $$



      The adjoint of $int_0^x$ is $int_x^1$. And the adjoint of $M_x$ (multiplication by $x$) is $M_x$. So $T$ is selfadjoint because
      begin{align}
      T &= left(int_0^xright)M_x+M_xleft(int_x^1right)\ &=left(int_0^xright)M_x+M_x^*left(int_0^xright)^* = T^*.end{align}






      share|cite|improve this answer









      $endgroup$


















        1












        $begingroup$

        $T$ is a Hibert-Schmidt operator because $min{x,y} in L^2([0,1]times[0,1])$. $Tf$ may be written as
        begin{align}
        Tf & = int_{0}^{1}K(x,y)f(y)dy \
        & = int_{0}^{1}min{x,y}f(y)dy \
        & = int_0^xyf(y)dy+xint_x^1 f(y)dy
        end{align}

        If $Tf=lambda f$ for some $fin L^2$ and $lambdainmathbb{C}setminus{0}$, then the above implies that $Tf$ is equal a.e. to a continuous function on $[0,1]$. Hence, $f$ is equal a.e. to a continuous function. So $Tf$ is continuously differentiable. So, assume without loss of generality, that $f$ is continuous. Then $(Tf)(0)=0$ and $(Tf)'$ exists with
        $$
        lambda f'= (Tf)'=xf(x)-xf(x)+int_x^1f(y)dty=int_x^1 f(y)dt
        $$

        So $f$ is $C^2$, $f(0)=0$, $f'(1)=0$ and $lambda f''=-f$ for every eigenfunction with eigenvalue $lambdane 0$. The eigenfunctions with non-zero eigenvalues are, therefore, constant multiplies of
        $$
        f_n = sin(npi x/2),;;; n=1,3,5,7,cdots, \
        lambda_n = frac{2}{npi}.
        $$



        The adjoint of $int_0^x$ is $int_x^1$. And the adjoint of $M_x$ (multiplication by $x$) is $M_x$. So $T$ is selfadjoint because
        begin{align}
        T &= left(int_0^xright)M_x+M_xleft(int_x^1right)\ &=left(int_0^xright)M_x+M_x^*left(int_0^xright)^* = T^*.end{align}






        share|cite|improve this answer









        $endgroup$
















          1












          1








          1





          $begingroup$

          $T$ is a Hibert-Schmidt operator because $min{x,y} in L^2([0,1]times[0,1])$. $Tf$ may be written as
          begin{align}
          Tf & = int_{0}^{1}K(x,y)f(y)dy \
          & = int_{0}^{1}min{x,y}f(y)dy \
          & = int_0^xyf(y)dy+xint_x^1 f(y)dy
          end{align}

          If $Tf=lambda f$ for some $fin L^2$ and $lambdainmathbb{C}setminus{0}$, then the above implies that $Tf$ is equal a.e. to a continuous function on $[0,1]$. Hence, $f$ is equal a.e. to a continuous function. So $Tf$ is continuously differentiable. So, assume without loss of generality, that $f$ is continuous. Then $(Tf)(0)=0$ and $(Tf)'$ exists with
          $$
          lambda f'= (Tf)'=xf(x)-xf(x)+int_x^1f(y)dty=int_x^1 f(y)dt
          $$

          So $f$ is $C^2$, $f(0)=0$, $f'(1)=0$ and $lambda f''=-f$ for every eigenfunction with eigenvalue $lambdane 0$. The eigenfunctions with non-zero eigenvalues are, therefore, constant multiplies of
          $$
          f_n = sin(npi x/2),;;; n=1,3,5,7,cdots, \
          lambda_n = frac{2}{npi}.
          $$



          The adjoint of $int_0^x$ is $int_x^1$. And the adjoint of $M_x$ (multiplication by $x$) is $M_x$. So $T$ is selfadjoint because
          begin{align}
          T &= left(int_0^xright)M_x+M_xleft(int_x^1right)\ &=left(int_0^xright)M_x+M_x^*left(int_0^xright)^* = T^*.end{align}






          share|cite|improve this answer









          $endgroup$



          $T$ is a Hibert-Schmidt operator because $min{x,y} in L^2([0,1]times[0,1])$. $Tf$ may be written as
          begin{align}
          Tf & = int_{0}^{1}K(x,y)f(y)dy \
          & = int_{0}^{1}min{x,y}f(y)dy \
          & = int_0^xyf(y)dy+xint_x^1 f(y)dy
          end{align}

          If $Tf=lambda f$ for some $fin L^2$ and $lambdainmathbb{C}setminus{0}$, then the above implies that $Tf$ is equal a.e. to a continuous function on $[0,1]$. Hence, $f$ is equal a.e. to a continuous function. So $Tf$ is continuously differentiable. So, assume without loss of generality, that $f$ is continuous. Then $(Tf)(0)=0$ and $(Tf)'$ exists with
          $$
          lambda f'= (Tf)'=xf(x)-xf(x)+int_x^1f(y)dty=int_x^1 f(y)dt
          $$

          So $f$ is $C^2$, $f(0)=0$, $f'(1)=0$ and $lambda f''=-f$ for every eigenfunction with eigenvalue $lambdane 0$. The eigenfunctions with non-zero eigenvalues are, therefore, constant multiplies of
          $$
          f_n = sin(npi x/2),;;; n=1,3,5,7,cdots, \
          lambda_n = frac{2}{npi}.
          $$



          The adjoint of $int_0^x$ is $int_x^1$. And the adjoint of $M_x$ (multiplication by $x$) is $M_x$. So $T$ is selfadjoint because
          begin{align}
          T &= left(int_0^xright)M_x+M_xleft(int_x^1right)\ &=left(int_0^xright)M_x+M_x^*left(int_0^xright)^* = T^*.end{align}







          share|cite|improve this answer












          share|cite|improve this answer



          share|cite|improve this answer










          answered Jan 5 at 23:19









          DisintegratingByPartsDisintegratingByParts

          59.2k42580




          59.2k42580























              0












              $begingroup$

              Since we have the integral identity
              $$ g:= Tf = int_0^x yf(y) dy + xint_x^1 f(y) dy $$
              Then $gin H^1$, and since $g' = int_x^1 f(y) dy$, actually $g in H^2$, with
              $$ -g'' = f$$
              So we actually have a Poisson equation, but with boundary conditions $g(0)=0,g(1) = int_0^1 yf(y) dy$. By setting $tilde g = g - x int_0^1 yf(y) dy$, we notice that we are equivalently looking for the weak solutions in $H^1_0$ to the Poisson equation with Dirichlet boundary conditions
              $$ -tilde g'' = f text{ on } (0,1),quad tilde g(0)=tilde g(1)=0,quad fin L^2 $$
              So if you already knew that the solution operator $fmapsto tilde g$ for this 1D Poisson equation was compact($H^1_0 subsetsubset L^2)$ and self-adjoint, we're done (the difference $ g-tilde g= xint_0^1 y f(y) dy$ is finite rank and self-adjoint).






              share|cite|improve this answer









              $endgroup$


















                0












                $begingroup$

                Since we have the integral identity
                $$ g:= Tf = int_0^x yf(y) dy + xint_x^1 f(y) dy $$
                Then $gin H^1$, and since $g' = int_x^1 f(y) dy$, actually $g in H^2$, with
                $$ -g'' = f$$
                So we actually have a Poisson equation, but with boundary conditions $g(0)=0,g(1) = int_0^1 yf(y) dy$. By setting $tilde g = g - x int_0^1 yf(y) dy$, we notice that we are equivalently looking for the weak solutions in $H^1_0$ to the Poisson equation with Dirichlet boundary conditions
                $$ -tilde g'' = f text{ on } (0,1),quad tilde g(0)=tilde g(1)=0,quad fin L^2 $$
                So if you already knew that the solution operator $fmapsto tilde g$ for this 1D Poisson equation was compact($H^1_0 subsetsubset L^2)$ and self-adjoint, we're done (the difference $ g-tilde g= xint_0^1 y f(y) dy$ is finite rank and self-adjoint).






                share|cite|improve this answer









                $endgroup$
















                  0












                  0








                  0





                  $begingroup$

                  Since we have the integral identity
                  $$ g:= Tf = int_0^x yf(y) dy + xint_x^1 f(y) dy $$
                  Then $gin H^1$, and since $g' = int_x^1 f(y) dy$, actually $g in H^2$, with
                  $$ -g'' = f$$
                  So we actually have a Poisson equation, but with boundary conditions $g(0)=0,g(1) = int_0^1 yf(y) dy$. By setting $tilde g = g - x int_0^1 yf(y) dy$, we notice that we are equivalently looking for the weak solutions in $H^1_0$ to the Poisson equation with Dirichlet boundary conditions
                  $$ -tilde g'' = f text{ on } (0,1),quad tilde g(0)=tilde g(1)=0,quad fin L^2 $$
                  So if you already knew that the solution operator $fmapsto tilde g$ for this 1D Poisson equation was compact($H^1_0 subsetsubset L^2)$ and self-adjoint, we're done (the difference $ g-tilde g= xint_0^1 y f(y) dy$ is finite rank and self-adjoint).






                  share|cite|improve this answer









                  $endgroup$



                  Since we have the integral identity
                  $$ g:= Tf = int_0^x yf(y) dy + xint_x^1 f(y) dy $$
                  Then $gin H^1$, and since $g' = int_x^1 f(y) dy$, actually $g in H^2$, with
                  $$ -g'' = f$$
                  So we actually have a Poisson equation, but with boundary conditions $g(0)=0,g(1) = int_0^1 yf(y) dy$. By setting $tilde g = g - x int_0^1 yf(y) dy$, we notice that we are equivalently looking for the weak solutions in $H^1_0$ to the Poisson equation with Dirichlet boundary conditions
                  $$ -tilde g'' = f text{ on } (0,1),quad tilde g(0)=tilde g(1)=0,quad fin L^2 $$
                  So if you already knew that the solution operator $fmapsto tilde g$ for this 1D Poisson equation was compact($H^1_0 subsetsubset L^2)$ and self-adjoint, we're done (the difference $ g-tilde g= xint_0^1 y f(y) dy$ is finite rank and self-adjoint).







                  share|cite|improve this answer












                  share|cite|improve this answer



                  share|cite|improve this answer










                  answered Jan 6 at 0:00









                  Calvin KhorCalvin Khor

                  11.8k21438




                  11.8k21438






























                      draft saved

                      draft discarded




















































                      Thanks for contributing an answer to Mathematics Stack Exchange!


                      • Please be sure to answer the question. Provide details and share your research!

                      But avoid



                      • Asking for help, clarification, or responding to other answers.

                      • Making statements based on opinion; back them up with references or personal experience.


                      Use MathJax to format equations. MathJax reference.


                      To learn more, see our tips on writing great answers.




                      draft saved


                      draft discarded














                      StackExchange.ready(
                      function () {
                      StackExchange.openid.initPostLogin('.new-post-login', 'https%3a%2f%2fmath.stackexchange.com%2fquestions%2f3063087%2fintegral-operator-in-l2%23new-answer', 'question_page');
                      }
                      );

                      Post as a guest















                      Required, but never shown





















































                      Required, but never shown














                      Required, but never shown












                      Required, but never shown







                      Required, but never shown

































                      Required, but never shown














                      Required, but never shown












                      Required, but never shown







                      Required, but never shown







                      Popular posts from this blog

                      Human spaceflight

                      Can not write log (Is /dev/pts mounted?) - openpty in Ubuntu-on-Windows?

                      File:DeusFollowingSea.jpg